หัวข้อ: ข้อสอบ 7th TMO
ดูหนึ่งข้อความ
  #41  
Old 13 พฤษภาคม 2010, 20:03
Aquarious Aquarious ไม่อยู่ในระบบ
เริ่มฝึกวรยุทธ์
 
วันที่สมัครสมาชิก: 25 ธันวาคม 2009
ข้อความ: 12
Aquarious is on a distinguished road
Default

ข้อ 6 วันที่ 2

Lemma $n\nmid r^{n-1}+1$ เมื่อ r เป็นจำนวนเฉพาะ

พิสูจน์ lemma

สมมุติว่ามี n ซึ่ง$n\mid r^{n-1}+1$

เขียน $n=p_1^{a_1}p_2^{a_2}...p_k^{a_k}$

เขียน ${p_i}-1=2^{m_i}q_i $ เมื่อ $q_i $เป็นจำนวนคี่

เลือก $p_i$ ที่ทำให้ $m_i=min ({m_1,m_2,...,m_k})$

ดังนั้น $n\equiv 1 (mod 2^{m_i}) (\because m_i คือmin)$


$\therefore n-1=2^{m_i}t$

$r^{2^{m_i}t}\equiv -1 (mod p_i )$

จาก $q_i$ เป็นคี่

ทำให้$ r^{(2^{m_i}q_i)t}\equiv -1 (mod p_i )$

$ r^{(p_i-1)t}\equiv -1 (mod p_i )$

จากแฟรมาจะได้ว่า $1\equiv -1 (mod p_i )$ เกิดข้อขัดแย้งเพราะ $p_i \not= 2$

ดังนั้น $n\nmid r^{n-1}+1$

กลับมาที่โจทย์

จากโจทย์ กำหนดว่า $pq\mid r^{p}+r^{q}$

ดังนั้น $p\mid r^{p}+r^{q}$

$r^q\equiv -r^p (mod p)$

$\therefore r^q\equiv -r (mod p)(\because จากแฟรมา)$

$ r^{pq}\equiv -r^p (mod p) (ยกกำลัง p และจาก p เป็นจำนวนคี่ )$

$ \therefore r^{pq}\equiv -r (mod p)$

ทำให้ $p\mid r^{pq}+r$

แต่ p เป็นจำนวนเฉพาะ และ p>r

$\therefore p\mid r^{pq-1}+1$

ทำนองเดียวกันจะได้ $q\mid r^{pq-1}+1$

จาก $(p,q)=1$

$\therefore pq\mid r^{pq-1}+1$

ซึ่งจาก Lemma จะได้ว่าไม่มี p q ที่สอดคล้อง

$\therefore ไม่มี p, q, r ที่สอดคล้องกับโจทย์$
ตอบพร้อมอ้างอิงข้อความนี้